Prove a Laplace Transform Equality

Click For Summary
The discussion revolves around proving a specific equality involving Laplace transforms. The initial attempt involved integrating an expression derived from the definition of the Laplace transform, but the user encountered difficulties with variable substitution and integration techniques. A suggestion was made to verify algebraic steps and to consider a theorem that simplifies the process, which states that under certain conditions, the Laplace transform of a function divided by t can be expressed as an integral of its transform. The user expressed frustration as this problem was the last hurdle in completing their homework. The conversation highlights the complexities of Laplace transforms and the potential for using theorems to simplify calculations.
Italo Campoli
Messages
7
Reaction score
0

Homework Statement



Prove that http://www4f.wolframalpha.com/Calculate/MSP/MSP26931c1531g07285beh7000062h7f6g1ggd95eea?MSPStoreType=image/gif&s=5&w=98.&h=38. =http://www4f.wolframalpha.com/Calculate/MSP/MSP6901c153574d0bdbh20000048829f0g4d1fi1d0?MSPStoreType=image/gif&s=5&w=69.&h=35.

The Attempt at a Solution



Using the Def of Laplace i got to ∫[e ^ -st - e ^ -t(s-1)] / t dt ; of course with limits from 0 to infinite

tryed then to do a variable change of w = st but i got inmmerse on a huge process of solving

∫{ [e ^ -w . e ^ (1/s-1)w] / w/s } dw/s , i know that the change was wrong from that point and i have been trying to solve it for parts or another change but haven't got across any luck with that so far

Id appreciate some help or any advice in how to proceed from that point, its the only problem left to finish my homework and its driving me crazy :S

Side Note: i haven´t seen yet Inverse Laplace, but a fellow Math Degree friend told me that using that i might solve it pretty easy.
 
Last edited by a moderator:
Physics news on Phys.org
Italo Campoli said:

Homework Statement



Prove that http://www4f.wolframalpha.com/Calculate/MSP/MSP26931c1531g07285beh7000062h7f6g1ggd95eea?MSPStoreType=image/gif&s=5&w=98.&h=38. =http://www4f.wolframalpha.com/Calculate/MSP/MSP6901c153574d0bdbh20000048829f0g4d1fi1d0?MSPStoreType=image/gif&s=5&w=69.&h=35.

The Attempt at a Solution



Using the Def of Laplace i got to ∫[e ^ -st - e ^ -t(s-1)] / t dt ; of course with limits from 0 to infinite

I would first check your algebra in multiplying e-st by e-t / t
 
Last edited by a moderator:
You don't have to use the formal definition of a Laplace transform because there is a theorem that will greatly simplify the calculation.

Theorem: Suppose ##f(t)## is piecewise continuous and of exponential order. By exponential order, I mean there exists a constant ##\alpha## such that ##e^{- \alpha t} |f(t)| \to 0## as ##t \to \infty##.

Suppose further ##\mathcal{L} \{ f(t) \} = F(s)## for ##s > c \geq 0##, and ##\displaystyle \lim_{t \to 0^+} \frac{f(t)}{t}## exists. Then:

$$\mathcal{L} \left \{ \frac{f(t)}{t} \right \} = \int_s^{\infty} F(x) \space dx, \quad s > c$$
 
  • Like
Likes Italo Campoli
Question: A clock's minute hand has length 4 and its hour hand has length 3. What is the distance between the tips at the moment when it is increasing most rapidly?(Putnam Exam Question) Answer: Making assumption that both the hands moves at constant angular velocities, the answer is ## \sqrt{7} .## But don't you think this assumption is somewhat doubtful and wrong?

Similar threads

  • · Replies 1 ·
Replies
1
Views
1K
  • · Replies 10 ·
Replies
10
Views
2K
  • · Replies 4 ·
Replies
4
Views
3K
  • · Replies 3 ·
Replies
3
Views
3K
  • · Replies 3 ·
Replies
3
Views
2K
Replies
2
Views
2K
  • · Replies 3 ·
Replies
3
Views
1K
  • · Replies 1 ·
Replies
1
Views
1K
  • · Replies 2 ·
Replies
2
Views
2K
  • · Replies 4 ·
Replies
4
Views
3K